LSAT and Law School Admissions Forum

Get expert LSAT preparation and law school admissions advice from PowerScore Test Preparation.

 Administrator
PowerScore Staff
  • PowerScore Staff
  • Posts: 8916
  • Joined: Feb 02, 2011
|
#81269
Complete Question Explanation

The correct answer choice is (B).

Answer choice (A):

Answer choice (B): This is the correct answer choice.

Answer choice (C):

Answer choice (D):

Answer choice (E):

This explanation is still in progress. Please post any questions below!
 gwlsathelp
  • Posts: 93
  • Joined: Jun 21, 2020
|
#78898
How do you approach this?
User avatar
 KelseyWoods
PowerScore Staff
  • PowerScore Staff
  • Posts: 1079
  • Joined: Jun 26, 2013
|
#79000
Hi gwlsathelp!

First, you have to make sure you have the correct Global setup for this game. Your completed Global diagram should look something like this:

L        N               K/               N
S     K/H               J             H/M
T     H/K               /K           M/H
          X                 Y                 Z

(If you have questions about the Global diagram, ask them here: https://forum.powerscore.com/lsat/viewtopic.php?t=11892)

Since this is a Local question, we want to create a mini-diagram to incorporate the new Local rule that is being introduced. In this case, we are asked what would happen if K was the leader for one of the committees. The only committee that K can be a leader of is Y (N is always the leader of the other 2 committees). If K is the leader of Y, then H is the only volunteer left who can be the treasurer of Y. So our local diagram looks like this:

L        N                 K               N
S     K/H               J             H/M
T     H/K               H            M/H
          X                 Y                 Z

The question then asks us "for which of the committees is the assignment of volunteers to positions fully determined?" According to our local diagram, committee Y is the only committee in which the assignment of volunteers to position is fully determined. In committee X, we don't know whether K or H is secretary or treasurer, and in Z we don't know whether M or H is secretary or treasurer. So the answer is (B).

Hope this helps!

Best,
Kelsey
 gwlsathelp
  • Posts: 93
  • Joined: Jun 21, 2020
|
#79010
Wow!! Thank you, Kelsey! This was fantastic.
 mollylynch
  • Posts: 62
  • Joined: Jul 21, 2023
|
#103245
Why can't M be the treasurer for Y?
 Adam Tyson
PowerScore Staff
  • PowerScore Staff
  • Posts: 5153
  • Joined: Apr 14, 2011
|
#103288
It's because M must serve on committee Z only, Molly. Here's why:

J and K cannot serve on Z.

Z needs three members.

That leaves only H, M, and N to fill that committee.

M serves on only one committee. Thus, M cannot serve on either X or Y, because M is committed to serving on Z.

If that's not still clear, try coming up with a complete solution to the game with M on committee Y as you suggested. Make sure you follow all the rules! You'll find that it cannot be done.
 mollylynch
  • Posts: 62
  • Joined: Jul 21, 2023
|
#103591
Thank you, I didn't catch that inference!

Get the most out of your LSAT Prep Plus subscription.

Analyze and track your performance with our Testing and Analytics Package.